ChaseDream
搜索
返回列表 发新帖
查看: 12215|回复: 4
打印 上一主题 下一主题

lsat-22-3-24

[复制链接]
楼主
发表于 2004-12-18 16:30:00 | 只看该作者

lsat-22-3-24

24. Ditrama is a federation made up of three autonomous regions. Korva. Mitro, and Guadar, Under the federal revenue-sharing plan, each region receives a share of federal revenues equal to the share of the total population of Ditrama residing in that region as shown by a yearly population survey. Last year the percentage of federal revenues Korva received for its share decreased somewhat even though the population survey on which the revenue-sharing was based showed that Korva's population had increased.


If the statements above are true, which one of the following must also have been shown by the population survey on which last year's revenue-sharing in Dirama was based? (A) Of the three regions Korva had the smallest number of residents (B) The population of Korva grew by a smaller percentage than it did in previous years (C) The populations of Mitro and Guadar each increased by a percentage that exceeded the percentage by
which the population of Korva increased. (D) Of the three regions. Korva's numerical increase in population was the smallest (E) Korva's population grew by a smaller percentage than did the population of at least one of the other two autonomous regions.


都没有人问的问题,我问,这个e怎么出来的,我知道数学公式分子分母的腿的出来但是如何在考场上用逻辑分析快速解答呢/???

沙发
发表于 2004-12-21 00:41:00 | 只看该作者
common sense la
板凳
 楼主| 发表于 2004-12-21 20:05:00 | 只看该作者

        鄙视我?!. 


NO SENSE 


[此贴子已经被作者于2004-12-21 20:12:54编辑过]
地板
发表于 2004-12-24 00:21:00 | 只看该作者

Should math be used in LSAT, it will not be difficult.

Upon you finish reading the stimulus, you should have known that: revenue hare decrease------- population share decrease. the difference between the absolute number and the percentage is the only math you need to know here.

Don't use Chinese math mind to do a question of American math

5#
发表于 2019-8-15 17:07:10 | 只看该作者
entia 发表于 2004-12-18 16:30
24. Ditrama is a federation made up of three autonomous regions. Korva. Mitro, and Guadar, Under the ...

Spot the question type: Must be true ( Necessary )

1. D = K + M + G

2. FR ( D ) = K/D + M/D + G/D

K increased; however, FR( K/D ) decreased why ?

Well, what if the population increased by K this year is less than either M or G ?

Lets say, If K = 5, M = 500, G = 400. and FR( K/D ) = 5/905

And now, If K =6, M = 1000, G = 400, for sure FR( K/D ) = 6/1406

Apparently, one of the regions must have a greater population increase than K.

E is the best answer.
您需要登录后才可以回帖 登录 | 立即注册

Mark一下! 看一下! 顶楼主! 感谢分享! 快速回复:

所属分类: 法学院申请

近期活动

正在浏览此版块的会员 ()

手机版|ChaseDream|GMT+8, 2024-4-27 07:24
京公网安备11010202008513号 京ICP证101109号 京ICP备12012021号

ChaseDream 论坛

© 2003-2023 ChaseDream.com. All Rights Reserved.

返回顶部